A lusty product

Calculus Level 3

n = 2 n 3 1 n 3 + 1 \large \displaystyle \prod_{n=2}^{\infty} \dfrac{n^3-1}{n^3+1} If the value of above product is in the form a b \dfrac{a}{b} , where a a and b b are coprime positive integers, find a + b a+b .


The answer is 5.

This section requires Javascript.
You are seeing this because something didn't load right. We suggest you, (a) try refreshing the page, (b) enabling javascript if it is disabled on your browser and, finally, (c) loading the non-javascript version of this page . We're sorry about the hassle.

5 solutions

Rishabh Jain
Jan 19, 2016

Using a 3 + b 3 = ( a + b ) ( a 2 a b + b 2 ) \small{\color{#3D99F6}{a^3+b^3=(a+b)(a^2-ab+b^2)}} P = n = 2 m ( n 1 ) ( n 2 + n + 1 ) ( n + 1 ) ( n 2 n + 1 ) P= \displaystyle \prod_{n=2}^{m} \dfrac{(n-1)(n^2+n+1)}{(n+1)(n^2-n+1)} = ( n = 2 m n 1 n + 1 ) × ( n = 2 m n 2 + n + 1 n 2 n + 1 ) =(\displaystyle \prod_{n=2}^{m} \dfrac{n-1}{n+1})\times (\displaystyle \prod_{n=2}^{m} \dfrac{n^2+n+1}{n^2-n+1}) = ( 1 3 × 2 4 × 3 5 × 4 6 × . . . . m 1 m + 1 ) × ( 7 3 × 13 7 × 21 13 × 31 21 × . . . m 2 + m + 1 m 2 m + 1 ) =(\dfrac{1}{3}\times \dfrac{2}{4}\times \dfrac{3}{5}\times \dfrac{4}{6}\times....\dfrac{m-1}{m+1})\times (\dfrac{7}{3}\times \dfrac{13}{7}\times \dfrac{21}{13}\times \dfrac{31}{21}\times...\dfrac{m^2+m+1}{m^2-m+1}) = 2 ( m 2 + m + 1 ) 3 ( m ) ( m + 1 ) =\dfrac{2(m^2+m+1)}{3(m)(m+1)} When m , P approaches 2 3 \rightarrow \infty,\color{forestgreen}{P~\text{approaches}~\dfrac{2}{3}}

Hence, 2 + 3 = 5 \Large\color{goldenrod}{2+3=5}

This is a misleading "solution", to put it politely. You make it look like n = 2 n 1 n + 1 = 2 \prod_{n=2}^{\infty}\frac{n-1}{n+1}=2 and n = 2 n 2 + n + 1 n 2 n + 1 = 1 3 \prod_{n=2}^{\infty}\frac{n^2+n+1}{n^2-n+1}=\frac{1}{3} .

Otto Bretscher - 5 years, 4 months ago

Log in to reply

Frankly speaking-Is that not so?? Can you please explain further why we cannot write it like that??

Rishabh Jain - 5 years, 4 months ago

Log in to reply

No, it isn't so at all. Think about it: n 1 n + 1 < 1 \frac{n-1}{n+1}<1 for all n n , so, how could the product come out to be 2? You have to find the partial products n = 2 m n 3 1 n 3 + 1 \prod_{n=2}^{m}\frac{n^3-1}{n^3+1} , expressed in terms of m , m, and then take the limit as m m goes to infinity.

The third line of your solution comes out to be 0 × 0\times \infty , so, you cannot break the limit up like this.

Otto Bretscher - 5 years, 4 months ago

Log in to reply

@Otto Bretscher Can I edit my solution like this: First bracket is 2 m ( m + 1 ) \dfrac{2}{m(m+1)} while second one is m 2 + m + 1 3 \dfrac{m^2+m+1}{3} and when m \rightarrow \infty product is 2 3 \dfrac{2}{3} .

Rishabh Jain - 5 years, 4 months ago

Log in to reply

@Rishabh Jain Right! As the end, write " P P approaches 2 3 \frac{2}{3} " to make a clean distinction between the partial product P P and its limit.

Otto Bretscher - 5 years, 4 months ago

Log in to reply

@Otto Bretscher Done... Now its complete I think.... :)

Rishabh Jain - 5 years, 4 months ago

You're posting fantastic solutions.Maybe you'll soon leave me behind in upvotes :P (+1)

Rohit Udaiwal - 5 years, 4 months ago

Log in to reply

Latex makes maths beautiful.... :)

Rishabh Jain - 5 years, 4 months ago

Why can't we use compounendo and dividendo

Rajath Kaimal - 5 years, 4 months ago
Oliver Piattella
Jan 22, 2016

Let's consider the general term a n n 3 1 n 3 + 1 = ( n 1 ) ( n 2 + n + 1 ) ( n + 1 ) ( n 2 n + 1 ) = ( n 1 ) + n ( n 1 ) ( n + 1 ) ( n + 1 ) + n ( n 1 ) ( n + 1 ) = 1 + 1 n ( n + 1 ) 1 + 1 n ( n 1 ) . a_n \equiv \frac{n^3-1}{n^3+1} = \frac{(n-1)(n^2+n+1)}{(n+1)(n^2-n+1)} = \frac{(n-1)+n(n-1)(n+1)}{(n+1)+n(n-1)(n+1)} = \frac{1 + \frac{1}{n(n+1)}}{1 + \frac{1}{n(n-1)}}\;. Let's call: x n 1 + 1 n ( n 1 ) . x_n \equiv 1 + \frac{1}{n(n - 1)}\;. We can write the original product as n = 2 a n = n = 2 x n + 1 x n . \prod_{n=2}^{\infty}a_n = \prod_{n=2}^{\infty}\frac{x_{n+1}}{x_n}\;. Numerators and denominators of consecutive terms therefore simplify. The partial product P N n = 2 N x n + 1 x n = x N + 1 x 2 = 1 + 1 N ( N + 1 ) 1 + 1 2 ( 2 1 ) = 2 3 [ 1 + 1 N ( N + 1 ) ] . P_N \equiv \prod_{n=2}^N\frac{x_{n+1}}{x_n} = \frac{x_{N+1}}{x_2} = \frac{1 + \frac{1}{N(N + 1)}}{1 + \frac{1}{2(2 - 1)}} = \frac{2}{3}\left[1 + \frac{1}{N(N + 1)}\right]\;. Thus lim N P N = 2 3 . \lim_{N\to\infty}P_N = \frac{2}{3}\;.

Chew-Seong Cheong
Jan 19, 2016

Consider:

n = 2 n 3 1 n 3 + 1 = n = 2 m ( n 1 ) ( n 2 + n + 1 ) ( n + 1 ) ( n 2 n + 1 ) = 1 ˙ ˙ 3 ˙ 2 ˙ 1̸3 ˙ ˙ ˙ 2̸1 ˙ 1̸3 ˙ ˙ 3̸1 ˙ 2̸1 ˙ ˙ 4̸3 ˙ 3̸1 ˙ . . . = 2 3 \begin{aligned} \prod_{n=2}^\infty \frac{n^3-1}{n^3+1} & = \prod_{n=2}^m \frac{(n-1)(n^2+n+1)}{(n+1)(n^2-n+1)} \\ & = \small \frac{1 \dot{} \color{#3D99F6}{\not{7}}}{\color{#D61F06}{\not{3}} \dot{}3} \dot{} \frac{2 \dot{} \color{#3D99F6}{\not{13}}}{\color{#D61F06}{\not{4}} \dot{} \color{#3D99F6}{\not{7}}} \dot{} \frac{\color{#D61F06}{\not{3}} \dot{} \color{#3D99F6}{\not{21}} }{\color{#D61F06}{\not{5}} \dot{} \color{#3D99F6}{\not{13}}} \dot{} \frac{\color{#D61F06}{\not{4}} \dot{} \color{#3D99F6}{\not{31}} }{\color{#D61F06}{\not{6}} \dot{} \color{#3D99F6}{\not{21}}} \dot{} \frac{\color{#D61F06}{\not{5}} \dot{} \color{#3D99F6}{\not{43}} }{\color{#D61F06}{\not{7}} \dot{} \color{#3D99F6}{\not{31}}} \dot{} ... \\ & = \frac{2}{3} \end{aligned}

a + b = 2 + 3 = 5 \Rightarrow a + b = 2 + 3 = \boxed{5}

Ramiel To-ong
Jan 20, 2016

nice problem.

Tom Engelsman
Jan 19, 2016

An old classic Putnam problem......Question A1 from the '77 Exam!

Very popular problem indeed. I have seen this a couple of time on Brilliant only.

Abhishek Sharma - 5 years, 4 months ago

0 pending reports

×

Problem Loading...

Note Loading...

Set Loading...